0 Daumen
389 Aufrufe

Hallo, ich habe solche Aufgabe bekommen. Kann jemand mir den Schritt zeigen und erklären? Vielen Dank

$$\sum \limits_{k=1}^{n}\frac{1}{k^2} \leq 2 - \frac{1}{n} $$

Avatar von

3 Antworten

0 Daumen
 
Beste Antwort

Hallo Rara,

ja - bei dieser Umformung muss man wissen wo man hin will.

Induktionsanfang sollte nicht das Problem sein. Mache ich mal für \(n=1\) und \(n=2\):$$\sum_{k=1}^1 \frac 1{k^2} = 1 \le 1 = 2 - \frac 11 \space \checkmark\\ \sum_{k=1}^2 \frac 1{k^2} = \frac 54 \le \frac 32 = 2 - \frac 12 \space \checkmark$$Nun der Übergang von \(n\) nach \(n+1\):$$\begin{aligned}\sum_{k=1}^{n+1}\frac{1}{k^2} &= \sum_{k=1}^{n}\frac{1}{k^2} \space + \frac 1{(n+1)^2} \\ & \le 2 - \frac 1n + \frac 1{(n+1)^2} \\ &\lt 2 - \frac 1n + \frac 1{n(n+1)} \\ &= 2 - \left( \frac{n+1}{n(n+1)} - \frac 1{n(n+1)} \right) \\ &= 2 - \frac{n}{n(n+1)} \\ &= 2 - \frac 1{n+1}\end{aligned}$$was zu beweisen war.

Avatar von 48 k

Hallo Werner-Salomon,

ja ich bin damit schon versucht, aber bei mir sieht es so :

$$\sum \limits_{k=1}^{n} \frac{1}{k^2} (n=1) : 1 \leq 2-1$$ das passt schon

und dann

$$\sum \limits_{k=1}^{n+1} \frac{1}{k^2} = 2-\frac{1}{n}+\frac{1}{(n+1)^2}$$ habe ich am Ende $$\leq 2 - \frac{1}{n+1} - \frac{2n+1}{n(n+1)^2}$$ bekommen. jetzt weiß ich nicht wie man $$- \frac{2n+1}{n(n+1)^2}$$ entfernen kann. Ist meine zweite Formel übrigens falsch?

Grüße

Rara

Hallo Rara,

das ist falsch: \(\sum \limits_{k=1}^{n+1} \frac{1}{k^2} \colorbox{#ffff00}{=} 2-\frac{1}{n}+\frac{1}{(n+1)^2}\) es ist nicht gleich, sondern kleiner oder gleich. Genauer: kleiner, wenn \(n \gt 1\) ist.

Der nächste Term ist richtig.

jetzt weiß ich nicht wie man \(- \frac{2n+1}{n(n+1)^2}\)entfernen kann.

Das ist doch gerade der Witz mit dem \(\le\) ! dort steht:$$ \dots \le 2 - \frac 1{n+1} - a$$und Du musst doch nur zeigen dass$$2 - \frac 1{n+1} - a \le 2 - \frac 1{n+1} $$ist! Und dies ist immer der Fall, wenn \(a \ge 0\) ist. Mehr nicht. Du kannst den Ausdruck hinter dem Minuszeichen getrost weglassen, wenn er größer oder gleich 0 ist. Und das ist sicher der Fall, da \(n \gt 0\). Klaro?

Gruß Werner

Hallo Werner-Salomon,

ja das war ein Tippfehler mit dem '=' Zeichnen.

2−1/(n+1)−a ≤ 2−1/(n+1) usw.

ohaaa!! das verstehe ich jetzt. Vielen Dank für die Erklärung !

Grüß

Rara

0 Daumen

Beginne mit dem Induktionsanfang. Wie sieht der bei dir aus?

Avatar von 53 k 🚀

hallo abakus

$$\sum \limits_{k=1}^{n}\frac{1}{k^2} \leq 2 - \frac{1}{n}$$ I.A (n=1) : 1 <= 2-1 . also hier ist richtig

$$\sum \limits_{k=1}^{n+1}\frac{1}{k^2} \leq 2 - \frac{1}{n}$$ I.V = 2 - 1/(n + 1)/(n+1)^2

ich habe diese schon berechnet und dann habe ich am Ende 2- 1/(n+1) - 2n+1 / (n+(n+1)^2). Ich weiß nicht wie man - 2n+1 / (n+(n+1)^2) entfernen kann.

Grüße

Rara

Sieh dir die übersichtliche Lösung von Werner an.

0 Daumen

Hallo

1. n=1 einsetzen, 1/1^2<=2-1/1 richtig. 2. Vors

=1 1/^2≤2−1/

dann Behauptung für n+1 Summe bis n+1= Summe bis n +1=Summe bis n+1/(n+1)^2  und das weiss man ist <=1-1/n+1/(n+1)^2 jetzt nur zeigen dass  das <1-1/(n+1)

Gruß lul

Avatar von 106 k 🚀

Hallo,

ja ich habe mit der gleichen Formel versucht aber bekomme ich am Ende ≤2−1/(n+1)−2n+1n/(n+1)^2. also nicht genau wie es in der Aufgabe steht. Ich weiß nicht wie man −2n+1n/(n+1)^2 entfernen kann.

Grüße

Rara

Ein anderes Problem?

Stell deine Frage

Willkommen bei der Mathelounge! Stell deine Frage einfach und kostenlos

x
Made by a lovely community